Đến nội dung

fecma21 nội dung

Có 362 mục bởi fecma21 (Tìm giới hạn từ 03-05-2020)



Sắp theo                Sắp xếp  

#140242 một bài cơ bản

Đã gửi bởi fecma21 on 05-01-2007 - 11:09 trong Bất đẳng thức và cực trị

bài toán

cho $\ \sum\limits_{i=1}^{n} a_{i}^2 + (\sum\limits_{i=1}^{n} a_{i} )^2 =101 $

CMR $\ |a_{i}| \leq 10 $ với mọi i

---------------------------------------------------------

Cho $\ a_{1}^2+a_{2}^2+...a_{{n}}^2+(a_{1}+a_{2}+...+a_{{n}})^2 =101 $

CMR $\ |a_{i}| \leq 10 $ với mọi i

( bài này hơi yếu 101 có thể thay = $\ \leq 121 $



#140114 một bài hệ khó

Đã gửi bởi fecma21 on 04-01-2007 - 16:28 trong Phương trình - hệ phương trình - bất phương trình

đây là bài thi tỉnh toán HD ; Giải pt sau ' $\ x = \sqrt{3} + \dfrac{x}{\sqrt{x^2-1}} $ ' :)

việc giải pt này cũng khá cơ bản ( có lẽ chỉ cần kt lớp 9 )

và bây giờ là một bài toán đấu tiên ' Giải hệ pt ' được xây dựng từ pt (*)

bài toán 1 Giải hệ pt sau

$\ x = \sqrt{3} + \dfrac{y}{\sqrt{z^2-1}} $

$\ y = \sqrt{3} + \dfrac{z}{\sqrt{x^2-1}} $

$\ z = \sqrt{3} + \dfrac{x}{\sqrt{y^2-1}} $

xin mời tất cả các bác ( ai giải hay hơn mình gọi = sư phụ hi hi )

( có thể TQ cho số $\ sqrt{3} $ = k cũng được )

ngoài ra mình cũng muốn trao đổi với các bạn về một bài toán sau

rõ ràng là một bài trongđề thi mà lời giải các sách every đều sai mà mình cũng không thể giải trực tiếp chỉ có thể chỉ

ra số nghiệm và CM thôi :Rightarrow

bài toán 2 giải hệ pt

$\ x.(z^2-4z+5) = 6 $

$\ y.(x^2-4.x+5) = 6 $

$\ z.(y^2-4y+5) = 6 $



#140110 Bất đẳng thức ko biết đã đưa lên chưa

Đã gửi bởi fecma21 on 04-01-2007 - 16:14 trong Phương trình, Hệ phương trình Lượng giác

bài này dễ quá và mình cũng chẳng cần dùng cái chi trâu bò cả

nếu a,b,c có một cái < 0 hay > 120 thì max < 2 ; ta xét tất cả thuộc (0,180)

chỉ cần cái này thôi $\ sinA+sinB+sinC \leq \dfrac{3.\sqrt{3}}{2} $ với $\ A+B+C = 180 $

từ đó ta có $\ sinx + siny+sinz \leq \dfrac{3.\sqrt{3}}{2} $ vớiu mọi x,y,z thỏa mãn x+y+z = 180

còn với x+y+z = 360 => (180-x)+(180-y)+(180-z) = 180 và sin(180-x) = sinx ;

ta chuyển về bài toán có X+Y+Z = 180 ; tim max sinX+sinY+sinZ ? OK



#140106 pt hay -> hệ pt khó

Đã gửi bởi fecma21 on 04-01-2007 - 16:03 trong Phương trình - Hệ phương trình - Bất phương trình

đây là bài thi tỉnh toán HD ; Giải pt sau ' $\ x = \sqrt{3} + \dfrac{x}{\sqrt{x^2-1}} $ ' :Rightarrow

việc giải pt này cũng khá cơ bản ( có lẽ chỉ cần kt lớp 9 )

và bây giờ là một bài toán đấu tiên ' Giải hệ pt ' được xây dựng từ pt :)

bài toán Giải hệ pt sau

$\ x = \sqrt{3} + \dfrac{y}{\sqrt{z^2-1}} $

$\ y = \sqrt{3} + \dfrac{z}{\sqrt{x^2-1}} $

$\ z = \sqrt{3} + \dfrac{x}{\sqrt{y^2-1}} $

xin mời tất cả các bác ( ai giải hay hơn mình gọi = sư phụ hi hi )

( có thể TQ cho số $\ sqrt{3} $ = k cũng được )

ngoài ra mình cũng muốn trao đổi với các bạn về một bài toán sau

rõ ràng là một bài trongđề thi mà lời giải các sách every đều sai mà mình cũng không thể giải trực tiếp chỉ có thể chỉ

ra số nghiệm và CM thôi (*)

bài toán 2 giải hệ pt

$\ x.(y^2-4y+5) = 6 $

$\ y.(z^2-4.z+5) = 6 $

$\ z.(x^2-4x+5) = 6 $



#138512 Cực trị lợong giác

Đã gửi bởi fecma21 on 18-12-2006 - 15:38 trong Công thức lượng giác, hàm số lượng giác

bài này dễ mà thầy



MINCOPSKI -> ..... + CÔ SI với cái này



#138061 các anh đừng quên chủ đề này nhé

Đã gửi bởi fecma21 on 16-12-2006 - 15:23 trong Phương trình - hệ phương trình - bất phương trình

mathmath post nhầm chủ đề à ? 3 cái VT là 3 cái BT bậc 2 > 0 => x,y,z>0

anh mà mod bên này là anh xóa rồi :Rightarrow :Leftrightarrow



#138060 Pt nhiều cách đây!

Đã gửi bởi fecma21 on 16-12-2006 - 15:19 trong Phương trình - hệ phương trình - bất phương trình

Bài Làm: Ta có:
http://dientuvietnam...x.cgi?(2006-x^2)^2=2006-x


Vậy phương trình có bốn nghiệm:

:Rightarrow :Leftrightarrow :Rightarrow :Rightarrow

hha ha thực ra cái cách này trông có vẻ trâu bò chứ thực ra nó là cái hướng của ducpbc chứ còn chi nữa ?

từ cái hệ trừ về => (x-y).(x+y)=x-y => (x-y).(x+y-1)=0 =>

<=>



#137367 Xin giải dùm 1 số bài toán tìm giới hạn

Đã gửi bởi fecma21 on 12-12-2006 - 20:03 trong Dãy số - Giới hạn

bạn học latex đi nhé ; không khó nhìn lắm ; học latex dễ mà

nếu cứ để thế này sẽ gây sự khó chịu cho bạn đọc và cũng khó dịch đề

mình góp ý như vậy



#137323 Thử sức nào!

Đã gửi bởi fecma21 on 12-12-2006 - 17:03 trong Bất đẳng thức và cực trị

bài này quen rôi mà



=>



#137314 Dễ thôi!

Đã gửi bởi fecma21 on 12-12-2006 - 16:45 trong Bất đẳng thức và cực trị

bài này đã có từ .... khá lâu thế mà chả có ai đả động gì nhỉ ?

bài này cũng khá hay mà ( có rất nhiều cách )

nhưng mạnhtuan thiếu đk là x,y,z thỏa mãn nhé

nào mời các bạn .



#133424 Một bài thi tỉnh

Đã gửi bởi fecma21 on 26-11-2006 - 09:36 trong Bất đẳng thức và cực trị

cám ơn thảo nhiều nhưng mình đang lo lắm

chẳng biết thế nào nữa :phi ^_^ ^_^

nếu đỗ bạn sẽ khao

( hì hì đây là chuyện ngoài lề các mod thông cảm )

em sẽ đóng to pic này tại đấy :D :phi



#133420 a^n+b^n>=1/2^n-1

Đã gửi bởi fecma21 on 26-11-2006 - 09:26 trong Bất đẳng thức và cực trị

bài này có thể dùng nhiều cách CM , và khá quen rồi

mục đích chủ yếu là CM cái BDT này

cái này có thể cm = quy nạp và dùng cái này

có thể cm = CÔ SI như bác quán vũ nhưng mà khó nhìn lắm , em làm lại như sau

từ a+b=1 => (2a)+(2b) = 2 BDT cần cm là <=>

BÀI TOÁN VỀ CMR :

BY CÔ SI : ..... => đfcm

ngoài ra có thể dùng JENSEN nhưng mà thôi

mình đóng topic nhé ^_^



#133217 GIỚI HẠN DÃY SỐ!

Đã gửi bởi fecma21 on 25-11-2006 - 16:12 trong Phương trình - Hệ phương trình - Bất phương trình

định lí STOLZ

Xét 2 dãy với



khi đó có



#132966 Một bài thi tỉnh

Đã gửi bởi fecma21 on 24-11-2006 - 16:43 trong Bất đẳng thức và cực trị

bài toán cho a,b,c>0 và

CMR :

3 dòng :cry :cry :cry

làm nhiều mà ngu , thi không làm được :geq :leq :geq -> dốt :in :alpha



#131365 giới hạn dãy số hay

Đã gửi bởi fecma21 on 19-11-2006 - 09:34 trong Phương trình - Hệ phương trình - Bất phương trình

bài toán

cho dãy xác định như sau :



TÍNH


hay hay hay hay hay



#131360 2 bài đạo hàm

Đã gửi bởi fecma21 on 19-11-2006 - 09:30 trong Phương trình - hệ phương trình - bất phương trình

chủ yếu là dùng lagrange và rolle

bài toán 1

cho

với mọi

CMR: cũng > 0 với mọi x

bài toán 2

cho đa thức f(x) có 3 nghiệm ;

Tìm số nghiệm của các hàm số sau :

1,

2,

3,

Có thể TQ cho n nghiêm



#131354 Một bài khá hay!

Đã gửi bởi fecma21 on 19-11-2006 - 09:16 trong Hàm số - Đạo hàm

với cách của manhtuan :) :vdots => cũng không tuần hoàn

vănchanh à , cách bạn phải cần 3 pt ; nữa



#131348 Bài học LG

Đã gửi bởi fecma21 on 19-11-2006 - 09:06 trong Công thức lượng giác, hàm số lượng giác

nhưng đó đâu phải = lượng giác :) :vdots

một bài nữa nè , trong mọi tam giác ABC



hì hì bài này mình nghĩ mãi mới ra có ai ccos cách không dùng hàm số mà chơi BDT thông thường thì oánh lên nhé



#131186 Có vẻ hơi quen

Đã gửi bởi fecma21 on 18-11-2006 - 16:30 trong Phương trình - Hệ phương trình - Bất phương trình

cái này dùng nội suy Niu tơn cm hệ số hữu tỉ

xét

xét tại n+1 điểm nguyên thì



theo đồng nhất hệ số suy ra hữu tỉ

sau đó => nguyên



#130847 Phép " chứng minh SGK .....sai !?"

Đã gửi bởi fecma21 on 17-11-2006 - 16:17 trong Dãy số - Giới hạn

bạn nhầm một sai lầm ' không thể chấp nhận ' là lấy tích các lim của biểu thức không hữu hạn .

có 2 t/c :

và cho cả số thực ...



#130568 lượng giác hay và ấn tượng

Đã gửi bởi fecma21 on 16-11-2006 - 16:38 trong Công thức lượng giác, hàm số lượng giác

cái bài này học đt thầy NG cho mà em vẫn chưa giải được?

bác vị thần sức mạnh post lời giải lên đi ? em đang rất cần đấy ....

bác giải gấp nhé =)) ;)) :in :geq



#130563 một bài cũng hay

Đã gửi bởi fecma21 on 16-11-2006 - 16:32 trong Bất đẳng thức và cực trị

bài toán 1

cho

CMR:

trước hết là bài 1 :

CÁCH 1 lượng giác

đặt a=tgA ; b=tgB => c=tgC ; BDT <=>

chuyển vế sang OK ....

cách 2 BDT thông thường

từ gt => đặt b,c tương tự => a+b+c = 1 => .....

các bác tự giải tiếp nào ??



#130560 HAI BÀI BẤT ĐẲNG THƯC KHÓ QUÁ

Đã gửi bởi fecma21 on 16-11-2006 - 16:24 trong Phương trình - hệ phương trình - bất phương trình

bài 2 với x,y>0 ; Ta cm : M >1 ;

thật vậy nếu x hoặc y >1 => đfcm;

TH x,y <1 đặt => m,n nguyên dương



=>

bằng BDT phụ ta có

cộng lại có đfcm ;



#130559 BẤT ĐẲNG THỨC GIAI THỪA!

Đã gửi bởi fecma21 on 16-11-2006 - 16:20 trong Bất đẳng thức và cực trị

các BDT bạn đưa ra là hơi lỏng đó

bạn có thể tham khảo các bài toán BDT số học như trên trong quyển

Bất đẳng thức số học của thầy PHAN HUY KHẢI nhé



#130361 MIN ?

Đã gửi bởi fecma21 on 15-11-2006 - 20:26 trong Bất đẳng thức và cực trị

các bài toán dạng này đã có rất nhiều và đã được viết chuyên đề về

phương pháp hệ số bất định

ở box THCS cũng có nè

http://diendantoanho...?showtopic=7908

Topic này anh khóa nhé